Can't finish equation solving step :p area between functions

  • Thread starter Thread starter franklingroye
  • Start date Start date
  • Tags Tags
    Area Functions
Click For Summary
The discussion revolves around finding the value of the constant m such that the area between the functions x^2 and mx in the first quadrant equals 8. The area is calculated using the integral of the difference between the two functions, leading to the expression (1/2)m^3 - (1/3)m^3. The user struggles with simplifying this equation to isolate m and equate it to 8. After some back-and-forth, they realize that the expression simplifies to (1/6)m^3 = 8, leading to the solution for m. The conversation highlights the importance of careful simplification in solving equations.
franklingroye
Messages
6
Reaction score
0

Homework Statement



The two functions are x2 and mx, where m is a positive constant. I'm asked to find the value for m where the region enclosed between the two equations in the first quadrant is equal to 8.

Homework Equations



n/a

The Attempt at a Solution



Since m is a constant, x2 and mx intersect at x = 0 and x = m. So therefore, the area between the equations would be:

(\frac{m}{2}x2 - \frac{1}{3}x3)|^{m}_{0}

Which simplifies to:

\frac{m}{2}m2 - \frac{1}{3}m3

So clearly all I need to do is set it equal to 8 and solve... however, I'm having major issues actually accomplishing this. I can't see how to get it so I just have m = (constant). Can anyone help? I know the answer I'm trying to get is ~3.63424 but no clue how to get there.
 
Physics news on Phys.org
m^3(1/2 - 1/3)=8
just get m^3 on one side by multiplying/dividing the equation by some appropriate number?
 
Oster said:
m^3(1/2 - 1/3)=8
just get m^3 on one side by multiplying/dividing the equation by some appropriate number?

My gosh, I didn't even notice that (m/2)(m^2) simplified to 1/2m^3. I feel so stupid now! Thank you :) I've been trying to solve it keeping the three different powers of m
 
no problem =)
 
Question: A clock's minute hand has length 4 and its hour hand has length 3. What is the distance between the tips at the moment when it is increasing most rapidly?(Putnam Exam Question) Answer: Making assumption that both the hands moves at constant angular velocities, the answer is ## \sqrt{7} .## But don't you think this assumption is somewhat doubtful and wrong?

Similar threads

Replies
8
Views
1K
  • · Replies 4 ·
Replies
4
Views
2K
  • · Replies 3 ·
Replies
3
Views
2K
  • · Replies 1 ·
Replies
1
Views
2K
  • · Replies 13 ·
Replies
13
Views
2K
  • · Replies 11 ·
Replies
11
Views
2K
Replies
3
Views
2K
  • · Replies 8 ·
Replies
8
Views
2K
  • · Replies 2 ·
Replies
2
Views
2K
  • · Replies 6 ·
Replies
6
Views
2K